Docsity
Docsity

Prepare for your exams
Prepare for your exams

Study with the several resources on Docsity


Earn points to download
Earn points to download

Earn points by helping other students or get them with a premium plan


Guidelines and tips
Guidelines and tips

English Summaries for Student, Schemes and Mind Maps of English Literature

Using for learning and practices

Typology: Schemes and Mind Maps

2024/2025

Uploaded on 06/26/2025

le-huy-11
le-huy-11 🇻🇳

2 documents

1 / 31

Toggle sidebar

This page cannot be seen from the preview

Don't miss anything!

bg1
Pass The TOEIC Test Introductory Course ANSWER KEY
34
LISTENING COMPREHENSION
PART 1 – Photographs
Try It Out
p3 Track 1
1. D In (A), taking sounds similar to talking, but there are no
customers in view. She is sitting, not standing, at the reception
desk (B). (C) is an assumption.
2. A (B) is confuses the similar-sounding stair and chair. They are
not waving at the window (C). The curtains are open (D).
3. D He’s examining a car, not a card (A). He’s fixing the tire, not the
car door (B). He is not driving the car (C).
4. B There are no tourists in the lake (A). (C) is incorrect because
snake sounds like lake. The trees are not under the water (D).
5. A In (B), trunk sounds similar to truck, but we cannot see the trunk
(B). He is sitting, not standing, in the truck (C). He is not getting
out of the truck (D).
6. C She is not closing the store (A) or cleaning the window (B). She
is looking at the floor not the door (D).
Improve Your Performance
Analyzing Distractors p8
Exercise B
1. D Corr ect. (A) Similar-sounding word (taking – talking). (B)
Incorrect key word (sitting not standing). (C) Incorrect
interpretation.
2. A Correct. (B) Similar-sounding word (stair – chair). (B) Incorrect
interpretation. (C) Incorrect key word (open not closed).
3. D Correct. (A) Similar-sounding word (card – car). (B) Incorrect
key word (tire not car door). (C) Incorrect interpretation.
4. B Correct. (A) Incorrect interpretation. (C) Similar-sounding word
(snake – lake). (D) Incorrect key word (near not under).
5. A Correct. (B) Similar-sounding word (trunk – truck). (C) Incorrect
key word (sitting not standing). (D) Incorrect interpretation.
6. C Correct. (A) Incorrect interpretation. (B) Incorrect key word
(floor not window). (D) Similar-sounding word (door – floor).
Similar-sounding Words p12
Exercise A Track 2
1. large, 2. hire, 3. rest, 4. bag, 5. wait, 6. frame, 7. choose, 8. nose,
9. sock, 10. waiter
Exercise B Track 3
1. The man is sitting at home. 2. He’s sailing a boat. 3. They’re walking
together. 4. She’s setting the table. 5. There’s a hat on the chair.
6. The woman is waiting for a train.
Exercise C Track 4
1. Incorrect (copy – coffee). 2. Correct. 3. Incorrect (feeding – reading
/ cook – book). 4. Incorrect (cable – table). 5. Correct. 6. Incorrect
(testing – resting).
Mini Test – Similar-sounding Words p13 Track 5
1. C She has a black hat, not a cat (A). There are leaves on the
ground, but they are not leaving the park (B). They are in the
park, not the dark (D).
2. C She’s looking at the drink, not the sink (A). She’s using the
coffee machine, not copy machine (B). She’s holding, not
folding, a paper cup (D).
3. B The word complaint (A) sounds like paint. The shelves are on
the right, not the light (C). The ladder, not the leader, is in the
middle (D).
Incorrect Key Words p14
Exercise A Track 6
1. False. She has short dark hair. 2. False. She’s wearing a skirt. 3.
True. 4. False. She’s standing near an escalator. 5. True. 6. True.
Exercise B Track 7
1. False. He’s sitting in a coffee shop. 2. True. 3. True. 4. False. The
spoon is on the saucer. 5. False. He’s texting on his phone. 6. True.
Mini Test – Incorrect Key Words p15 Track 8
1. C It’s light outside, not dark (A). People are sitting on sofas, not on
the floor (B). The blinds, not the windows, are half closed (D).
2. D There’s nobody on the scooter (A). The scooter is parked, not
lying, on the road (B). The L-sign is on the front, not the back, of
the scooter (C).
3. B She’s using a fax machine, not a computer (A). She’s holding a
document, not opening one (C). She’s wearing a short jacket,
not a long jacket (D).
Incorrect Interpretations p16
False Accounts Track 9
1. False. He is not leaning against the wall. 2. True. 3. True. 4. False.
There are notices on the board, but he’s not looking at them. 5.
False. It is not a vending machine. 6. False. He’s not holding a glass.
Unsupported Assumptions Track 10
1. Assumption. 2. Fact. 3. Assumption. 4. Fact. 5. Assumption.
6. Fact.
Mini Test – Incorrect Interpretations p17 Track 11
1. B T here is a music system on the table, but we can’t assume she’s
listening to it (A). She not typing at a desk (C). We can’t assume
she’s going to sit down (D).
2. D He may need help, but we can’t assume he is asking for help
(A). He could be delivering the box so (B) is an assumption.
There is a cart, but he is not sitting on it (C).
3. C We can’t assume a match is about to start (A), or that the
courts are closed (B). We can see tennis courts, but no signs of
construction (D).
Steps to Success
p18
Practice 1 C They are indoors, but they are not playing a game (A) or
Track 12 sitt ing in a coffee shop (B). We do not know what they
are discussing (D).
Practice 2 D Only one woman is wearing glasses (A). We cannot
Track 13 assume a purchase is being made (B). There is a
clock, not a lock, on the wall (C).
Practice 3 A He is in a restaurant, but he is not ordering a meal (B).
Track 14 He’s holding a teapot, not his glasses (C). He is
pouring, not drinking, tea (D).
Practice 4 D We can’t assume this is a competition (A). The man is
Track 15 hitting, not hiding, a tennis ball (B). Nobody is picking
up the balls (C).
Practice 5 B She is not washing her hair (A). The poster is on the wall,
Track 16 not on the mirror (D). They are not watching television
(C).
Review Test
p25 Track 17
1. C He is not filling the car with gas (A). He is loading the t runk, not
the truck (B). (D) is an assumption.
2. A T he food is on a table, not on the floor (B). Nobody is sitting on
the sofa (C). There is a teapot, not a saucepan, on the table (D).
3. B He standing on a ladder, not a chair (A). There is nobody sitting
outside (C). The chairs are facing away from the poster (D).
4. D She’s sitting behind the counter, not on it (A). (B) is an assumption.
There is a picture on the wall (C), but she is not looking at it.
5. C The bus has already arrived (A). The weather is sunny, with no
rain (B). They are boarding a bus, not a train (D).
6. A He is not putting food in an oven (B), or pouring water into the
sink (C). He is working in, not walking to, the kitchen (D).
Answer Key
pf3
pf4
pf5
pf8
pf9
pfa
pfd
pfe
pff
pf12
pf13
pf14
pf15
pf16
pf17
pf18
pf19
pf1a
pf1b
pf1c
pf1d
pf1e
pf1f

Partial preview of the text

Download English Summaries for Student and more Schemes and Mind Maps English Literature in PDF only on Docsity!

LISTENING COMPREHENSION

PART 1 – Photographs

Try It Out p3 Track 1

  1. D In (A), taking sounds similar to talking, but there are no customers in view. She is sitting, not standing, at the reception desk (B). (C) is an assumption.
  2. A (B) is confuses the similar-sounding stair and chair. They are not waving at the window (C). The curtains are open (D).
  3. D He’s examining a car, not a card (A). He’s fixing the tire, not the car door (B). He is not driving the car (C).
  4. B There are no tourists in the lake (A). (C) is incorrect because snake sounds like lake. The trees are not under the water (D).
  5. A In (B), trunk sounds similar to truck, but we cannot see the trunk (B). He is sitting, not standing, in the truck (C). He is not getting out of the truck (D).
  6. C She is not closing the store (A) or cleaning the window (B). She is looking at the floor not the door (D).

Improve Your Performance

Analyzing Distractors p Exercise B

  1. D Correct. (A) Similar-sounding word (taking – talking). (B) Incorrect key word (sitting not standing). (C) Incorrect interpretation.
  2. A Correct. (B) Similar-sounding word (stair – chair). (B) Incorrect interpretation. (C) Incorrect key word (open not closed).
  3. D Correct. (A) Similar-sounding word (card – car). (B) Incorrect key word (tire not car door). (C) Incorrect interpretation.
  4. B Correct. (A) Incorrect interpretation. (C) Similar-sounding word (snake – lake). (D) Incorrect key word (near not under).
  5. A Correct. (B) Similar-sounding word (trunk – truck). (C) Incorrect key word (sitting not standing). (D) Incorrect interpretation.
  6. C Correct. (A) Incorrect interpretation. (B) Incorrect key word (floor not window). (D) Similar-sounding word (door – floor). Similar-sounding Words p Exercise A Track 2
  7. large, 2. hire, 3. rest, 4. bag, 5. wait, 6. frame, 7. choose, 8. nose,
  8. sock, 10. waiter Exercise B Track 3
  9. The man is sitting at home. 2. He’s sailing a boat. 3. They’re walking together. 4. She’s setting the table. 5. There’s a hat on the chair.
  10. The woman is waiting for a train. Exercise C Track 4
  11. Incorrect (copy – coffee). 2. Correct. 3. Incorrect (feeding – reading / cook – book). 4. Incorrect (cable – table). 5. Correct. 6. Incorrect (testing – resting). Mini Test – Similar-sounding Words p13 Track 5
  12. C She has a black hat, not a cat (A). There are leaves on the ground, but they are not leaving the park (B). They are in the park, not the dark (D).
  13. C She’s looking at the drink, not the sink (A). She’s using the coffee machine, not copy machine (B). She’s holding, not folding, a paper cup (D).
  14. B The word complaint (A) sounds like paint. The shelves are on the right, not the light (C). The ladder, not the leader, is in the middle (D). Incorrect Key Words p Exercise A Track 6
  15. False. She has short dark hair. 2. False. She’s wearing a skirt. 3. True. 4. False. She’s standing near an escalator. 5. True. 6. True. Exercise B Track 7
    1. False. He’s sitting in a coffee shop. 2. True. 3. True. 4. False. The spoon is on the saucer. 5. False. He’s texting on his phone. 6. True. Mini Test – Incorrect Key Words p15 Track 8
    2. C It’s light outside, not dark (A). People are sitting on sofas, not on the floor (B). The blinds, not the windows, are half closed (D).
    3. D There’s nobody on the scooter (A). The scooter is parked, not lying, on the road (B). The L-sign is on the front, not the back, of the scooter (C).
    4. B She’s using a fax machine, not a computer (A). She’s holding a document, not opening one (C). She’s wearing a short jacket, not a long jacket (D). Incorrect Interpretations p False Accounts Track 9
    5. False. He is not leaning against the wall. 2. True. 3. True. 4. False. There are notices on the board, but he’s not looking at them. 5. False. It is not a vending machine. 6. False. He’s not holding a glass. Unsupported Assumptions Track 10
    6. Assumption. 2. Fact. 3. Assumption. 4. Fact. 5. Assumption.
    7. Fact. Mini Test – Incorrect Interpretations p17 Track 11
    8. B There is a music system on the table, but we can’t assume she’s listening to it (A). She not typing at a desk (C). We can’t assume she’s going to sit down (D).
    9. D He may need help, but we can’t assume he is asking for help (A). He could be delivering the box so (B) is an assumption. There is a cart, but he is not sitting on it (C).
    10. C We can’t assume a match is about to start (A), or that the courts are closed (B). We can see tennis courts, but no signs of construction (D).

Steps to Success p

Practice 1 C They are indoors, but they are not playing a game (A) or Track 12 sitting in a coffee shop (B). We do not know what they are discussing (D). Practice 2 D Only one woman is wearing glasses (A). We cannot Track 13 assume a purchase is being made (B). There is a clock, not a lock, on the wall (C). Practice 3 A He is in a restaurant, but he is not ordering a meal (B). Track 14 He’s holding a teapot, not his glasses (C). He is pouring, not drinking, tea (D). Practice 4 D We can’t assume this is a competition (A). The man is Track 15 hitting, not hiding, a tennis ball (B). Nobody is picking up the balls (C). Practice 5 B She is not washing her hair (A). The poster is on the wall, Track 16 not on the mirror (D). They are not watching television (C).

Review Test p25 Track 17

  1. C He is not filling the car with gas (A). He is loading the trunk, not the truck (B). (D) is an assumption.
  2. A The food is on a table, not on the floor (B). Nobody is sitting on the sofa (C). There is a teapot, not a saucepan, on the table (D).
  3. B He standing on a ladder, not a chair (A). There is nobody sitting outside (C). The chairs are facing away from the poster (D).
  4. D She’s sitting behind the counter, not on it (A). (B) is an assumption. There is a picture on the wall (C), but she is not looking at it.
  5. C The bus has already arrived (A). The weather is sunny, with no rain (B). They are boarding a bus, not a train (D).
  6. A He is not putting food in an oven (B), or pouring water into the sink (C). He is working in, not walking to, the kitchen (D).

Answer Key

  1. D We cannot tell whether they are for sale (A). There are no drivers in view (B), and it is not a traffic jam (C).
  2. B He’s lifting weights, not waiting in the gym (A). (C) is a false account. (D) is an assumption.
  3. C (A) is an assumption. There is a large notice, but it is not in the way (B). The word educator sounds like elevator (D).
  4. B The pillows are on the bed, not the floor (A). The picture is on the wall next to the bed, not above it (C). Nobody is sleeping in the bed (D).

PART 2 – Question-Response

Try It Out p31 Track 18

  1. B (A) repeats morning, but does not answer the question. (C) is a response to a suggestion.
  2. C (A) uses the same verb finish, but in the past simple. (B) repeats work, but as a verb and without answering the question.
  3. A (B) repeats going. (C) answers a different question (Are you going to see…?).
  4. C (A) answers a different question (What color is…?) and confuses blue with new. (B) repeats printer, but does not answer the question.
  5. B (A) confuses get back (return) with back (part of the body). (C) answers a different question (Why are you coming back…?).
  6. B (A) repeats like and confuses the similar-sounding words down and town. (C) answers a different question (Why don’t you want to sit down?).
  7. A (B) responds to a different statement (Is this your ticket?). (C) repeats return, but in a different context.
  8. C (A) uses soon to mislead recently. (B) refers to a different subject (He).
  9. A (B) repeats visit. (C) answers a different question (Would you like to visit…?).
  10. C (A) confuses the similar-sounding words called and cold. (B) uses the same context, but does not answer the question.
  11. B (A) confuses the similar-sounding words computer and producer. (C) answers a different question (Whose is that…?).
  12. C (A) confuses hat with that, and (B) confuses suitcase with shoelace.
  13. B (A) confuses the similar-sounding words feeling and healing. (C) does not answer the question.
  14. A The response Yes (B) cannot answer a Choice question. (C) confuses Eve with leave.
  15. C Neither (A) not (B) answers the question.
  16. B (A) confuses the similar-sounding words station and donation, and (C) confuses rush with bus.
  17. C (A) uses the same verb, but a different subject. (B) confuses meeting with seating.
  18. C (A) confuses task with ask, and (B) confuses partly with party.

Improve Your Performance

Wh- Questions p Exercise A

  1. When, 2. How, 3. What, 4. Who, 5. Which, 6. Why, 7. Where,
  2. Whose Exercise B
  3. f, 2. h, 3. g, 4. b, 5. a, 6. e, 7. c, 8. d Questions with What p Exercise A
  4. c, 2. e, 3. b, 4. f, 5. d, 6. a Exercise B
  5. C, 2. B, 3. C, 4. A, 5. C, 6. A Exercise C Track 19
  6. B (A) answers a different question (Who went to…?). (C) does not answer the question.
  7. A The subject is the same, but neither (B) not (C) answers the question. 3. B (A) confuses the similar-sounding words drink and think. (C) confuses won’t with want. 4. A (B) does not answer the question. (C) answers a different question (Will you sell your…?). 5. C (A) confuses the noun talk with the verb discuss, and does not answer the question. (B) answers a different question (Would you like to…?). 6. B (A) does not answer the question. (C) repeats desk, but answers a different question (Do you like that desk?). Mini Test – Questions with What p34 Track 20 1. C (A) answers a different question (Would you like to…?). (B) confuses by repeating time. 2. A (B) confuses gift with present from presentation. (C) answers a different question (Did the clients like…?). 3. B (A) confuses the noun watch with watching. (C) confuses tree with TV. 4. C (A) uses seat which sounds similar to eat. (B) confuses meet (which sounds like meat) with eat. 5. A (B) uses plane which sounds similar to plan. (B) confuses to pay with today. 6. B (A) repeats car, but answers a different question (Is that your car?). (C) confuses far with car. 7. C (A) confuses the similar-sounding words coast and cost. (B) confuses the similar-sounding words lost and cost). 8. B (A) confuses the similar-sounding words drink and think. (C) repeats think but does not answer the question. Questions with When or Where p Exercise A 1. d, 2. e, 3. b, 4. a, 5. c, 6. f Exercise B 1. B, 2. C, 3. B, 4. B, 5. A, 6. C Exercise C Track 21 1. C (A) answers the statement Here’s your black pen. (B) answers a different question (Is that a black pen?). 2. B (A) confuses the similar-sounding words hall and call. (C) answers a different question (Will you call?). 3. A (B) repeats interview, and (C) repeats place, but neither answers the question. 4. B (A) uses presentation which sounds similar to demonstration. (C) refers to time, but does not answer the question. 5. A (B) confuses short with report, and does not answer the question. (C) uses handle (meaning manage) which sounds like hand in. 6. B (A) uses dispute which sounds like suit. (C) answers a different question (How much was you suit?). Mini Test – Questions with When or Where p35 Track 22 1. B (A) answers a different question (Can I get a taxi?). (C) confuses tax with taxi. 2. C (A) uses art which sounds similar to start. (B) confuses moving with movie. 3. A (B) answers a different question (When was this machine made?). (C) confuses paid with made. 4. A (B) confuses take orders with place an order. (C) confuses place with replace. 5. B (A) uses heights which sounds similar to night. (C) does not answer the question. 6. C (A) does not answer the question. (B) confuses find it with founded. 7. A (B) uses looking which sounds similar to booking. (C) confuses the noun book with booking (meaning reservation). 8. B (A) confuses the similar-sounding words way and stay. (C) answers a different question (How long will we be at …?). Questions with Who, Whose, Why, or Which p Exercise A 1. b, 2. d, 3. f, 4. c, 5. a, 6. e
  1. B (A) repeats seat and uses file which sounds like aisle. (C) also repeats seat, but does not answer the choice question.
  2. A (B) repeats afternoon, but does not answer the question. (C) also fails to answer the question.
  3. B (A) misleads by using choice to relate to choose. (C) does not refer to the past, and does not answer the question.
  4. C (A) uses printer which might refer to print, but does not answer the question. (B) confuses colorful with color. Tag Questions p Exercise A
  5. d, 2. e, 3. a, 4. c, 5. f, 6. b Exercise B
  6. B, 2. A, 3. B, 4. A, 5. C, 6. B Exercise C Track 31
  7. A (B) uses thought which sounds similar to bought. (C) confuses print and again with printer and another.
  8. C (A) uses request which sounds similar to success. (B) uses lunch which sounds similar to launch.
  9. A (B) confuses fair with the similar-sounding word fares. (C) repeats train and the verb go, but does not answer the question.
  10. B (A) refers to a different subject. (C) uses clerk which sounds like work.
  11. C (A) uses merger which sounds similar to worker. (B) confuses walk with work.
  12. A (B) repeats boss and uses arrived now to confuse with late. (C) uses sell which sounds like tell, and confuses later with late. Mini Test – Tag Questions p41 Track 32
  13. A (B) uses the past tense. (C) uses station which sounds like vacation.
  14. A (B) does not answer the question. (C) uses unsure which sounds similar to brochures.
  15. B (A) uses home which sounds like phone. (C) does not answer the question.
  16. C (A) uses walk which sounds like work. (B) uses compartment which sounds similar to department.
  17. B (A) does not answer the question logically. (C) repeats anywhere, but does not answer the question.
  18. B (A) repeats they and job in the plural, but confuses the context. (C) misleads by using the same verb offer.
  19. A (B) answers a different question (Kenny’s gone to England, hasn’t he?). (C) does not answer the question.
  20. B (A) repeats paper, but with a different meaning. (C) uses note which sounds like wrote. Negative Questions p Exercise A
  21. d, 2. f, 3. c, 4. a, 5. e, 6. b Exercise B
  22. C, 2. A, 3. B, 4. B, 5. A, 6. C Exercise C Track 33
  23. B (A) answers a different question (When did you scan…?). (C) misleads by using scans to relate to scanner.
  24. A (B) confuses moon which with afternoon. (C) answers a different question (Where is Jen?).
  25. B (A) uses announcement which sounds similar to accountant. (C) confuses count on (meaning rely on) with the similar-sounding accountant.
  26. C (A) uses weaker which sounds like speaker. (B) uses spoke which relates to speaker, but does not answer the question.
  27. B (A) and (C) use similar-sounding words (think and sink sound similar to ink).
  28. C (A) repeats coat, but is an illogical answer. (B) uses wrote which sounds like coat. Mini Test – Negative Questions p42 Track 34
  29. B (A) repeats forward, but is an illogical response. (C) uses the past tense.
  30. C (A) and (B) use similar-sounding words (great and plate sound similar to late). 3. B (A) and (C) both refer to driving, but do not answer the question. 4. A (B) uses phone which sounds like home. (C) repeats Lisa and home, but is not a logical response. 5. B (A) confuses the subject. (C) does not answer the question. 6. C (A) uses overdone which sounds like fun. (B) is an illogical response. 7. C (A) refers to the context and repeats year, but does not answer the question. (B) is an illogical response. 8. B (A) uses enjoyed which sounds similar to join. (C) uses beginner which sounds like dinner. Embedded Questions p Exercise A 1. b, 2. e, 3. a, 4. c, 5. d, 6. f Exercise B 1. B, 2. B, 3. C, 4. B, 5. A, 6. B Exercise C Track 35 1. A (B) uses thank which sounds like bank. (C) answers a different question (When does the bank open?). 2. A (B) answers a different question (What time are we meeting?). (C) misleads by using restaurant which assumes eating for meeting. 3. B (A) is an illogical response. (C) uses knees which sounds like keys, with a singular subject. 4. C (A) and (B) use related words to shipped (ship and shipping), but neither answers the question. 5. B (A) uses collect which sounds similar to correct. (C) confuses stress with address. 6. A (B) uses tax which sounds like taxi. (C) repeats taxi, answers a different question (Will we take a taxi there?). Mini Test – Embedded Questions p43 Track 36 1. C (A) uses shoes which sounds like news. (B) does not answer the question. 2. B (A) uses up which sounds like cup. (C) answers a different question (Can I use your cup?). 3. B (A) uses Paul’s which sounds like walls. (C) colorful refers to color, but does not answer the question. 4. C (A) uses came which sounds like train. (B) repeats train, but does not answer the question. 5. C (A) repeats use, but does not answer the question. (B) is an illogical response. 6. A (B) answers a different question (Is Mr. Chen in his office?). (C) answers a different question (How old is Mr. Chen?). 7. A (B) uses nice which sounds like price. (C) uses guest which sounds like best. 8. B (A) misleads as are no job vacancies, only job cuts. (C) confuses the context, referring to cuts as in injuries. Statements p Exercise A 1.e, 2. b, 3. f, 4. a, 5. d, 6. c Exercise A 1. c, 2. f, 3. a, 4. d, 5. b, 6. e Exercise A 1. a, 2. c, 3. e, 4. b, 5. d, 6. f Exercise B 1. C, 2. C, 3. B, 4. C, 5. A, 6. C, 7. B, 8. B, 9. B, 10. A Exercise C Track 37 1. B (A) uses decide which sounds like outside. (C) refers to a different subject. 2. C (A) answers a different question (When does the post office close?). (B) misleads by using main office in place of post office. 3. B (A) uses purse which sounds like worse. (C) answers the question How is your headache?. 4. A (B) uses ate which sounds like late. (C) uses the future simple. 5. C (A) uses can which sounds like plan. (B) repeats plan. Both are illogical responses. 6. C (A) misleads by using the verb suit. (B) uses share which sounds like wear. 7. B (A) uses the past simple. (C) confuses raining with training.
  1. C (A) is an illogical response. (B) confuses lower with low in a different context.
  2. C (A) and (B) are both illogical responses.
  3. A (B) uses community which sounds similar to opportunity. (C) uses chance which is similar in meaning to opportunity, but the sentence is in the past simple. Mini Test – Statements p45 Track 38
  4. B (A) says where the lecture took place, and (C) says how long it lasted. Neither is a logical response to the statement.
  5. A (B) uses drink which sounds like ink. (C) is illogical as the paper supply is not the problem.
  6. C (A) repeats the time in a different way, but in the past simple. (B) uses thirsty which sounds like thirty.
  7. B (A) uses sit which relates to seat, and floor which sounds like door. (C) uses four which sounds like door, but does not answer the question.
  8. A (B) repeats car, but is not a logical response. (C) uses the phrase get lost which means lose your way rather than lose something.
  9. C (A) uses ship which sounds like trip. (B) confuses the adverb well with healthy.
  10. B (A) uses contact which sounds like contract. (C) uses the verb cancel, in the past simple, but begins Yes and therefore is not a logical response.
  11. B (A) uses collect which sounds like connect. (C) confuses the verb hold (meaning carry) with the meaning wait. Words with Multiple Meanings p Exercise A 1.a, 2. b, 3. b, 4. b, 5. a, 6. a, 7. a, 8. a Exercise B
  12. break, 2. book, 3. class, 4. full, 5. kind, 6. play, 7. address, 8. hot Mini Test – Words with Multiple Meanings p46 Track 39
  13. B (A) confuses the verb book (meaning reserve) with the noun. (C) does not answer the question.
  14. C (A) confuses the verb fix (meaning arrange) with the meaning repair. (B) repeats fire alarm, but is an illogical response.
  15. B (A) is an illogical response due to the verb use. (C) confuses the adjective hard (meaning solid or inflexible) with the meaning difficult.
  16. B (A) confuses the adjective hot (meaning spicy) with the meaning very warm. (C) refers to weather, not indoor temperature.
  17. A (B) confuses the adjective wrong (meaning incorrect) with the meaning unsuitable. (C) repeats wrong, but with the meaning dishonest or unethical rather than unsuitable.
  18. C Both (A) and (B) confuse the noun class (meaning group of students) with the meaning standard of travel.
  19. A (B) uses good meaning kind. (C) uses good meaning skilled or competent.
  20. A (B) uses play as a verb play music. (C) uses the noun play meaning show or theatrical performance. Similar-sounding Words p Exercise A
  21. can, plan, than, Japan, 2. well, hotel, personnel, bell, 3. sweet, meat, sheet, concrete, 4. word, bird, third, heard, 5. say, disobey, repay, delay Exercise B
  22. C – report, support, resort, 2. C – plane, remain, came, train,
  23. B – time, climb, nine, sign, 4. C – position, exhibition, competition, edition, 5. A – eight, late, straight, wait, 6. C – prize, fries, advised, surprise, 7. A – charge, large / promotion, lotion, 8. C – prefer, refer, transfer / nine, time, mind Mini Test – Similar-sounding Words p47 Track 40
  24. C (A) uses copies which sounds similar to coffee. In (B) believe sounds similar to leave.
  25. B (A) and (C) use similar-sounding words (checked and collect sound similar to project). 3. A (B) and (C) use similar-sounding words (design and mine sound similar to sign). 4. C (A) and (B) use similar-sounding words (hotel and sell sound similar to Personnel). 5. A (B) uses take which sounds similar to cake. In (C) read sounds similar to ready. 6. C (A) uses depart which sounds similar to chart. In (B) art sounds similar to chart. 7. B (A) is an illogical response. (C) uses the verb decide which sounds similar to applied. 8. B (A) uses play which sounds similar to (re)pay. In (C) unknown sounds similar to loan. Homophones p Exercise A 1. a, 2. b, 3. a, 4. b, 5. b, 6. a, 7. b, 8. a Exercise B 1. C – Where, wear, 2. C – hour, our, 3. A – read, red, 4. B – write, right, 5. B – sun, son, 6. B – knows, nose, 7. A – ate, eight, 8. C – see, sea Mini Test – Homophones p48 Track 41 1. C (A) confuses the adjective new with the past simple knew. (B) is an illogical response. 2. A (B) confuses the verb meet with the noun meat. (C) is an illogical response. 3. C (A) is an illogical response. (B) confuses the noun road with the past simple rode. 4. B (A) confuses the modal auxiliary would with the noun wood. (C) is an illogical response. 5. C (A) uses the contraction They’re and (B) uses the possessive pronoun their, which both sound like There. 6. B (A) confuses the noun hole with the adjective whole. (C) is an illogical response. 7. B (A) confuses the adjective sweet with the noun Suite. (C) is an illogical response. 8. A (B) is an illogical response. (C) confuses the verb stare with the noun stair.

Steps to Success p

Practice 1 Track 42: Warm-up: Would, Tuesday is fine/I’d prefer the 28th, Answer: C Practice Track 43: C Practice 2 Track 44: Warm-up: Do, Yes, usually/Not always, Answer: B Practice Track 45: B Practice 3 Track 46: Warm-up: What, He’s a businessman/He’s a student, Answer: A Practice Track 47: B Practice 4 Track 48: Warm-up: Can, I’ll try my best/Sorry, I’m busy, Answer: B Practice Track 49: B Practice 5 Track 50: Warm-up: I don’t want, You should leave now/ Neither do I, Answer: C Practice Track 51: A

Review Test p56 Track 52

  1. B (A) confuses living with leaving, and answers a different question (How long have you been living in Paris?). (C) answers a different question (Why are you…?).
  2. A (B) repeats milk, but does not answer the question. (C) answers a different question (Where is the …?).
  3. A (B) confuses the subject and meaning of working. (C) confuses the similar-sounding words commuters and computer.
  4. C (A) refers to a different subject, and confuses handle with canceled. (B) uses ordered which related to the noun order, but does not answer the question.
  5. B (A) answers a different question (What did you study at college?). (C) refers to the future.
  6. C (A) does not answer the question. (B) confuses So with snow and tonight with might.
  7. B (A) repeats take out, but as a verb. (C) uses the same verb get, but is an illogical response.
  1. D The woman says the vegetables will be ready soon. The man says he checked the steak. They are therefore not in a restaurant (A), or buying food (B). (D) cannot be inferred. Questions about the Location p65 Track 58
  2. A The woman asks for the man’s ticket. The man asks about a platform. (B) and (C) and (D) are therefore incorrect.
  3. D The man asks where the meats are. The woman replies near the fruit and vegetables, and mentions a sign. He is shopping in a supermarket. (A) and (B) are incorrect. (C) is not mentioned.
  4. C The woman says she needs to return these books. As they are overdue, the man says she may have to pay a fine. (A) and (B) are therefore incorrect. (D) is not mentioned.
  5. B The woman mentions a package and refers to bags of mail and stamps. The word scales is a distractor for weighing food items (A). (C) is not mentioned. The phrase hurt my back tries to confuse with health (D).
  6. C The man wants a car for a few days and the woman suggests a compact and looks for a price. The car is not being repaired in a garage (A), and the man is not buying a car (B). (D) is not mentioned.
  7. A The man complains of a toothache. The woman asks to take a look and says she needs to do something about this urgently. We can infer she is a dentist. (B), (C), and (D) cannot therefore be inferred. Mini Test – Listening for Main Ideas p66 Track 59
  8. B The woman asks about adult education classes. The word courses also indicates a college.
  9. B The woman asks about business courses. She is not looking for a job (A) or a vacation (C). The phrase twice a week for ten weeks refers to the class schedule, not a work schedule (D).
  10. C The man is giving information about the Business Administration course. He is not studying (A) or on vacation (B). (D) is a distractor, as the word business appears twice.
  11. A The woman mentions that this soup is cold and the bread is hard. There is food (B) and the man apologizes (C). The soup is cold, not the room (D).
  12. C Only a waiter, not a friend (A), would apologize for the cold soup. A cook (B) would be in the kitchen not waiting tables in the restaurant. (D) is not mentioned.
  13. D The man promises to change the soup and bring more bread. (A), (B), and (C) are not mentioned.
  14. B The woman says the annual conference in Paris is next week. She is not going there on vacation (A). (C) cannot be inferred and (D) is not mentioned.
  15. A The man says the hotels…will be full by now and the woman replies I’ve tried everywhere. She mentions taking a train, but not that a train is late (B). The hotels may be expensive (C), but that is not the problem. (D) cannot be inferred.
  16. C The woman says she will book a hotel further out (of the city). (A) is incorrect, (B) cannot be inferred, and (D) is not mentioned.
  17. C The man mentions Lisa’s birthday and suggests a party to celebrate. (A) is not mentioned. (B) distracts by suggesting food. They are making preparations, but not for a meeting (D).
  18. D The man says he will get some sandwiches delivered. The room they choose is easy to clean, but we cannot infer the man will clean it (A). The man will call to order food, not call home (B). The woman will order the drinks, but it is not mentioned that the man will have a drink (C).
  19. D They say the party will take place in a Meeting Room 4, and that people can stop by before they leave the office. We can infer they work together. Listening for Details Questions about Time p68 Track 61
    1. C The man says we should be able to finish it next week. The woman says See you on Monday (A) but this is a distraction. (B) is a distractor for have a good weekend. (D) relates to how long the project has lasted so far.
    2. D The woman says You only bought it two weeks ago. (A) is not mentioned. (B) repeats over the weekend. (C) is incorrect.
    3. D The woman says the lecture starts at three and lasts for two hours. (A) confuses the time with the length of the lecture. (B) is the start time. (C) is the time the man thinks the lecture finishes.
    4. A The woman says It’s only Tuesday and later promises I’ll do it right away. (B) is not mentioned. (C) is the day of the meeting. (D) is a distractor for weekend.
    5. C The man says he goes on Mondays and Fridays. (A) repeats every day in the woman’s question. The man mentions three times a week (B) but doesn’t go that often. (D) is incorrect.
    6. D The man advises the woman to do it right away when you get in tomorrow. (A) repeats right away but in the context of tomorrow. (B) is the current time. (C) is not mentioned. Questions about Reasons p69 Track 62
    7. C The man says A lot of documents seem to be missing. (A) is a distractor for the old system (i.e. the previous system). (B) is not mentioned. (D) refers to computer and confuses the context.
    8. B The man agrees it is convenient to be able to walk to work. (A) is incorrect, as his new home is smaller. (C) relates to train and walk but confuses the context. (D) relates to his previous home.
    9. D The man says their coworker James is away at a convention. (A) repeats main office but confuses the context. (B) confuses manage with manager. (C) repeats convention but refers to the woman.
    10. D The woman says I have to wait for a client to call. (A) is not mentioned. (B) confuses the Jenkins account with accounts. (C) repeats invoice but confuses the context.
    11. C The man says Mr. Fraser resigned, and a new Research Director is needed. (A) confuses resigned and declined. (B) repeats new but confuses the context. (D) refers to the length of time Mr. Fraser was in his job.
    12. A The man says he has a really bad stomachache. (B) repeats wait, but this refers to the man’s annual review. (C) confuses doctor and daughter. (D) repeats annual review but it isn’t the reason he can’t come in. Questions about Plans p70 Track 63
    13. C They decide to play tennis and go for a long walk. (A) confuses tennis and test. (B) is incorrect as the man says they finished that report. (D) repeats country but confuses the context.
    14. D The woman plans to apply for a permanent position and is in Toronto. Vancouver (A) is mentioned, but she says I’ve changed my mind. (B) repeats contract but confuses the context. They talk about her job (C) but she didn’t quit it – it ended.
    15. C The man asks Is a credit card OK? (A) confuses the verb check with the noun check. (B) repeats cash but he doesn’t have enough. (D) is not mentioned.
    16. A The woman mentions a blue dress and then later says the blue is a little more suitable. (B) and (C) are mentioned but are incorrect. (D) confuses suitable and suit.
    17. A The woman accepts a job offer. (B) is incorrect as she will give a month’s notice. She will be working, not on vacation (C). She has found a job, and is not looking for one (D).
    18. B The man mentions plans to close our Munich branch. There is already a branch in London (A). (C) repeats investors but confuses the context. The company will employ some people in New York, not lay off employees there (D).

Questions about Offers and Requests p71 Track 64

  1. A The man says he will get a sandwich and says I can get one for you too. He is going to buy food, not do the woman’s shopping (B). The woman asks the man to buy her a drink (C). She will give him some money, rather the man lend her some (D).
  2. C The man asks Would you mind staying late tonight... (A) repeats five o’clock. (B) repeats Monday but confuses the context. (D) relates to the woman’s plans for the evening.
  3. B The woman says I can give you my monitor. (A) and (D) are not mentioned. (C) repeats desk but confuses the context.
  4. A The man asks Would you like some tea or coffee? (B) is not mentioned, but late and wait sound like ate. The man offers to call Mrs. Aiten on her cell phone, not give it to the woman (C). (D) confuses check as a verb with check as a noun.
  5. B The man mentions a chair and asks Do you mind if I take this one? (A) is not mentioned. (C) repeats friends. (D) relates to any minute and confuses the context.
  6. C The woman asks for ten percent off all future orders. (A) repeats boss. (C) confuses competitive with competition. (D) is not mentioned. Questions about Suggestion and Advice p72 Track 65
  7. B The woman suggests the man look at job sites on the web. The man has already tried employment agencies (A) and newspapers (C). (D) is not mentioned.
  8. D The woman suggests the man find a good taxi company to collect Mr. Chan. (A) repeats Mr. Chan. (B) repeats schedule. (C) is not mentioned.
  9. B The man advises the woman to contact Media Relations and ask them about a vacancy there. (A) is incorrect as the woman doesn’t want to leave the company. (C) relates to qualified, but there is no mention of taking qualifications. (D) is not indicated.
  10. D The woman says the office is small and suggests moving to someplace bigger. (A) confuses the idea of a bigger office with expanding the company. The woman suggests the man move closer to the city, not live in it (B). Moving to a different city (C) is not mentioned.
  11. A The man says How about eating out and mentions a new Italian place across the street. (B) and (C) refer to what is usually done. (D) relates to the woman’s suggestion.
  12. A The man says I’d I were you, I’d catch a train to a nearby station. (B) refers to the woman’s original intention. The man says the train is quicker than the bus (C). (D) is not mentioned. Questions about Opinions p73 Track 66
  13. C The man complains The guy on the violin was terrible and the woman says she couldn’t hear the piano. (A) is not mentioned. (B) confuses couldn’t hear with loud music. (D) is incorrect as they were relieved it was only an hour.
  14. A The man says It’s so difficult to remember what goes where and refers to all these bins, implying the system is confusing. (B) is not mentioned. (C) and (D) cannot be inferred.
  15. D The woman says they will be lucky to reach the targets and she will be surprised if we can reach any of them. She feels the targets are very high already, not that they should be higher (A). (B) relates surprised to surprising. (C) is incorrect.
  16. A The woman says the shop is not marked in the leaflet and it took her ages to work out where it was. (B) is incorrect as the shop was finished last week. (C) confuses the problem of finding space in the museum with the size of the shop. (D) is not mentioned.
  17. D The man says the company will fail unless something is done, or they may not survive another six months. (A) is incorrect. (B) repeats lucky. (C) confuses a lot of decisions with divisions.
  18. A The woman says the suggested price is twenty percent more than our rivals and asks if anyone would buy it. She thinks there is a big difference in price, not that it is a little high (B). (C) and (D) are incorrect. Mini Test – Listening for Details p74 Track 67 1. A The man says I need to talk about his latest invoice. (B) confuses invoice and voice. (C) is not mentioned. (D) relates to Mr. Yang’s lunch meeting. 2. D The woman asks Would you like to leave a message? She does not offer to call the man (A). (B) and (C) are not mentioned. 3. C The woman says Mr. Yang should be out by two. (A) cannot be inferred. (B) confuses soon with noon. (D) is not mentioned. 4. A The woman asks Do you want to go for a meal? (B) confuses leaving for Houston and pack with a vacation. (C) repeats French, but this is the type of restaurant the woman mentions. The man needs to go home, not move home (D). 5. C He is going to Houston on business the next day. He will sign a contract, not rewrite one (A). (B) is not mentioned. (C) confuses pack with snack. 6. D The man is going home to pack. (A) is incorrect. (B) refers to his trip the next day. (C) relates to the woman’s request. 7. C The man says I have a couple of shirts I’d like you to wash. Various times are mentioned, but he is not checking the time (A). He wants someone to pick up his laundry from his room, not room service (B). (C) relates to a couple of (meaning two), but confuses shirts with desserts. 8. A The woman asks the man to put the shirts in a blue plastic bag. A pink form (B) is mentioned, but by the man. (C) repeats closet. (D) confuses closet and deposit. 9. B The woman says laundry collected before six P.M. should be back before ten the following morning. (A) is not mentioned. (C) refers to the current time. (D) refers to the deadline for laundry collection. 10. D The man mentions they are thirty minutes late. (A) is not mentioned. (B) repeats map, but the woman has not forgotten it. (C) confuses the context and also confuses on the left with left, the past tense of leave. 11. C The man says We were supposed to be here at three and that they are thirty minutes late. 12. B The man asks the woman to tell the reception desk we’re here. Making Inferences Questions about Attitude p76 Track 69 1. D The man sounds tired. He says he has driven for five hours, in heavy traffic, and It’s been raining the whole time. He does not mention food (A), or feeling sick (B), and (C) is not indicated. 2. A The woman sounds frustrated that the client canceled the order. She says she spent three days negotiating the deal, and gave them everything they asked for. (B), (C), and (D) cannot be inferred. Questions about Deductions p76 Track 70 1. B The woman questions whether the bill is correct, and says I’m amazed at the cost, adding that she doesn’t have enough cash to pay. We can deduce the bill is more than she anticipated. (A) and (C) are not mentioned.(D) cannot be inferred 2. A When asked if the Blue Kings are popular, the man says Are you kidding? which suggests they are well known. He also adds I can’t believe I got tickets, implying tickets soon sell out. We can deduce the band is popular. (B) is incorrect. (C) and (D) cannot be inferred. Mini Test – Making Inferences p76 Track 71 1. B The man complains it is too hot, and the woman says Jim is working on it as fast as he can. The man then says the whole system needs replacing. We can infer that it and the system relate to air conditioning. The windows aren’t broken (A), they simply don’t open. (C) and (D) cannot be inferred. 2. C We can tell from the man’s tone and the complaints he makes that the man feels angry. (A) is a distractor, referring to relax. (B) and (D) cannot be inferred. 3. B We can deduce from upgraded, first class, frequent flyer program and flight that they are talking about a journey by plane. (A) cannot be inferred. (C) confuses two meanings of the
  1. C The tests will start As of next month.
  2. D Lee says his calls is about the new computer system we’re designing. (A) confuses designing with resigning. (B) incorrectly assumes online search function means planning a website. (C) repeats new and uses design, but he is not starting a new company.
  3. A Lee says a lot more work is needed, and mentions financial implications. (B) is not mentioned. (C) cannot be inferred. (D) is incorrect as he says we haven’t included this in the cost.
  4. A Lee says I’m free next Tuesday. (B) repeats afternoon. (C) is not mentioned. (D) is incorrect as there are financial implications.
  5. B Mrs. Gifford has been Vice President here for three years. (A) refers to the position she will take up next month. The speaker is the Chairman (C). (D) is not mentioned.
  6. D The speaker says she first came to us as Vice President, three years ago.
  7. B The speaker asks the audience to join me in a round of applause to congratulate Mrs. Gifford. (A) assumes she is leaving the company. (C) repeats Chairman. (D) repeats position.
  8. D They are in a Space Museum. (A) incorrectly links books with a library. (B) is not mentioned. (C) relates the varied items the speakers lists with a department store.
  9. A The speaker is announcing that work is finally finished on the museum’s brand new gift shop. (B) confuses renovation with donation. The speaker does not thank anyone (C). Although she lists various products on sale, she is not advertising them (D).
  10. C The speaker says Shortly, the renovation of the cafeteria will also be completed. (A) confuses renovation with vacation. (B) relates to the speaker’s wish for strong sales. (D) repeats financial, but there is no suggestion of difficulties ahead.

Improve Your Performance

Listening for Main Ideas Questions about the Speaker p92 Track 80

  1. C The phrases Welcome aboard, flight AE564, flying, an altitude of, land, and flight all indicate this is in an airplane.
  2. C The speaker is welcoming the audience to the Seaview Bay Resort. The phrases check in at reception and Welcome Guest Pack suggest this is a hotel. The speaker says I’m in charge here.
  3. D The speaker mentions Joe’s Grill, steak, and dish, and refers to Everyone who works with me in the kitchen. Questions about the Topic p92 Track 81
  4. B The speaker mentions the man’s experience and knowledge, and invites him to attend the final stage of interviews. She is talking about a job vacancy.
  5. A The speaker mentions Washing your hands as it prevents the spread of disease. She reminds the audience to clean the tables, food trays, and the area around the beds.
  6. C The ad mentions 50 percent off everything and great end-of- the-season bargains for listeners who come to the store this weekend. Questions about the Audience p93 Track 82
  7. B The speaker mentions the next stop and says they will visit the Statue of Liberty, which is the last destination in our tour today.
  8. B The speaker refers to a half price promotion in the fruit and vegetable section. The audience are not working on a farm (A), selling food (C), or eating in a restaurant (D).
  9. C The message gives a number for callers to dial for emergency dental advice. Questions about the Location p93 Track 83
  10. D The speaker mentions the Impressionist rooms, and says there are over sixty paintings, mentioning Monet and Renoir.
  11. C The speaker says this train will arrive on Platform 2.
  12. B The key words mentioned are reports, grade, end-of-semester exam, study and students. Questions about the Purpose p94 Track 84 1. A The speaker says it is important to exercise regularly, and mentions the benefits of a daily walk. He also mentions cycling, golf and tennis. Serious health problems (B) are mentioned, and team sports (C), but neither is the main reason for the talk. The speaker says more exercise results in a longer life (D), but this is a result of exercise, not the purpose of the announcement. 2. C The speaker says he wants to express his gratitude for the staff’s hard work over the last year. They work at a conference venue (A), and have won an award, but he is not accepting it now (B). (D) is not mentioned. 3. B The woman is paying tribute to Dr. Williams, who is retiring after 37 years at the research facility. No new employees are mentioned (A). They work in a research facility, but she is not opening one (C). (D) confuses hard work with the need to work harder. Mini Test – Listening for Main Ideas p95 Track 85 1. B The ad tries to attract customers with free offers and big discounts. (A) and (C) are not mentioned. The checks will result in safer cars (D), but this is not the main purpose of the ad. 2. D The ad promises free engine and winter tire checks. No products are mentioned (A), only services. (B) confuses a warranty for work done on a car with vehicle insurance. Prices for services are reduced, but no cars are on sale (C). 3. C The ad is directed at all listeners; loyal customers and people who are new to Mike’s Motors. 4. B The report mentions a national strike will take place following the breakdown in contract negotiations between a farm workers union and management. 5. C The speaker is the host of a feature called Jobs Bulletin on WNRC Radio. She is a news presenter. 6. A The report is being made from the radio station. A union is mentioned but they are not outside the headquarters (B). The speaker mentions a press conference (C), but that was three hours ago. (D) refers to the union leader. 7. A The woman is preparing the audience for a talk by local historian Peter Fletcher. 8. C The speaker is mostly talking about Richard Warren, a settler who arrived in 1620. The topic is local history, but she is not speaking about its importance (A). She only introduces historian Peter Fletcher, rather than talks about him (B). The Mayflower is mentioned (D), but is not the topic. 9. B The talk is held at the Cape Cod Historical Society, and is for the general public. 10. D The woman is introducing Mr. Sanchez, who she says is the latest member of our team. 11. D The woman says Mr. Sanchez will work with everyone in the Sales and Marketing Department. (A), (B), and (C) cannot be inferred. 12. B The woman mentions main course and dessert, so this would indicate they are in a restaurant. Listening for Details Questions about Numbers, Dates, and Times p97 Track 87 A 1. C The payments were paid every month for the last six months. 2. C The speaker says the case will probably go to trial next month. 3. A Parker Buildings is the company mentioned. No other building companies are thought to be involved. B 1. B The speaker says the cafeteria will close for approximately one month. 2. A The Coffee Shop is on the third floor. 3. D Employees will each receive $50 in coupons per week.

Questions about Reasons and Requests p98 Track 88 A

  1. B The speaker says the delay is because of road repairs.
  2. B Passengers are advised to go to Bay 3 and wait there.
  3. D The next bus at Bay 3 is the 15:45 service to London, not Newcastle. B
  4. B The ad offer ultrafast broadband at the cheapest rates.
  5. A Advance Net Plus customers who have a 24-month contract can get a $50 credit. They are encouraged to sign up for two years.
  6. A New customers can get a free two-gigabyte memory stick, but only until the end of the month. Questions about People and Places p99 Track 89 A
  7. A The speaker says he will move to Sun City, Arizona.
  8. C His first job was as an Assistant Manager. He was then Section Director (A). He is now Vice President (D). (B) Is not mentioned.
  9. A The speaker says he will be handing over responsibilities to Mr. Hammond. B
  10. D Only graduate students can borrow up to ten books at a time.
  11. B The speaker says The red zone is the silent area and No talking in the red zone.
  12. B Books can be left in the book drop at the West entrance if the library is closed. Questions about Problems and Suggestions p100 Track 90 A
  13. C The company asked for $100,000, but the bank is only prepared to lend half that amount.
  14. B The speaker says that they might need to wait for bigger offices.
  15. D He says let’s make a list of our ideas for cutting costs. B
  16. A Kathy says I can’t get my work e-mail.
  17. D She says I don’t know when I’ll be able to use it again.
  18. A She asks please call me and gives a phone number. Questions about Plans and Sequences p101 Track 91 A
  19. B She begins by saying Before I begin by introducing our keynote speaker.
  20. B The talk on Innovation will take place in Room 403.
  21. A The presenter who will be introduced first is Dr. Fraser. B
  22. C The speaker says we have two weeks before the launch.
  23. D He mentions that starting today there will be a TV advertising campaign.
  24. D He is going to fly to Los Angeles at three o’clock. Mini Test – Listening for Details p102 Track 92
  25. A The message says For a billing inquiry, please press 3.
  26. C There is 50 percent discount on all headphones and speakers.
  27. D The message says to speak to a customer service representative, please press 6.
  28. C The ad mentions family fun and activities for children of all ages, and free meals for children under five.
  29. A The Aloha restaurant specializes in authentic local food.
  30. C Anyone booking online will get breakfast absolutely free.
  31. A The man says that the Regent Hotel is too small for us these days.
  32. C The Roosevelt is in the Adirondack Park, which is outside of the city.
  33. B He hands out the brochure and says let’s talk about it.
  34. C There are three tables of six all booked for between seven thirty and eight.
  35. B They are having a 50th birthday party.
  36. D The woman says I’ll go and tell the chef. We can infer she will go into the kitchen. Making Inferences Questions about Attitude p104 Track 94
    1. B The speaker sounds grateful, saying Thank you and later repeating her appreciation.
    2. A The man says the Jameson contract was something we all tried to win, and mentions the lucky woman who succeeded will get a huge bonus. His tone is jealous. Questions about Deductions p104 Track 95
    3. B The speaker asks employees who have parked there to move their cars as soon as possible.
    4. A The woman says the situation isn’t good, and that stocks are down again. She mentions orders have fallen. Mini Test – Making Inferences p104 Track 96
    5. A The speaker says we can’t wait any longer, and that they are ten minutes behind schedule.
    6. B Her tone is frustrated, especially towards the end that’s their problem.
    7. D The man apologizes that Carolina Strakovsky cannot perform, saying we understand that you came to hear Carolina Strakovsky, and offers a refund for those that want it.
    8. A We can infer that the man feels disappointed that the event can’t take place as planned.

Steps to Success p

Practice 1 Track 97 1 What/announced, 2 How long/Vicks/Mundo Medical, 3 What/implied/news

  1. C The announcer says the C.E.O. of Mundo Medical is leaving to join rival PRC Chemicals.
  2. C She has been there for more than twenty years.
  3. B The man refers to the announcement as shocking. Practice 2 Track 98 1 problem/website, 2 How long/problem, 3 What/Lee/do
  4. C The woman complains that there is still a wrong telephone number on the website.
  5. D She says It’s been almost four weeks now.
  6. B She wants Lee to do this ASAP, preferably today. Practice 3 Track 99 1 Where/announcement/made, 2 What/ passengers/do, 3 When/arrive/London
  7. C The words luggage, seat, aisles, racks, and bags are related, but buffet service and train indicate the location.
  8. A Passengers are asked to make sure your luggage is labeled correctly and to put small bags in the racks above your seat.
  9. C The man says We will be arriving in London at 9:15. (A) is the current time in Great Britain. (B) and (D) aren’t mentioned. Practice 4 Track 100 1 What/aim/message, 2 kind/movie/7:00 P.M., 3 Where/movie/rock music
  10. C The message is all about tonight’s features.
  11. B The romantic comedy Always a Bridesmaid is the only movie at 7:00 P.M.
  12. D This is where Metal Music Mania is showing. Practice 5 Track 101 1 Who/speaker/addressing, 2 How/Argo 250/ compare, 3 How long/guarantee/Argo 250
  13. A The Argo 250 is available at all good department stores, so the speaker is not addressing buyers (A). This is a sales pitch, not a board meeting (C). The reference to returns policy and shipping is a distractor for post office (D).
  14. A The speaker mentions the Argo 250 uses 50 percent of the electricity of other leading brands.
  15. D The vacuum has a lifetime warranty.

Review Test p112 Track 102

  1. C The announcement begins Attention Shoppers and mentions store, checkout counters, purchases, and food and drink.
  1. C Both (A) and (B) are illogical responses.
  2. A (B) confuses half and staff. (C) repeats say.
  3. C (A) answer a different question (How much is parking?). (B) repeats park, but as a noun.
  4. B (A) confuses male (relating this to women) with mail. (C) assumes they are waiting for delivery of an order, not mail.
  5. C (A) uses past tense of the verb work and the distractor fine. (B) confuses the subject and topic.
  6. A (B) confuses amazed with raise and refers to a different subject. (C) is an illogical response.
  7. B (A) answers a different question (How long was the flight?). (C) uses the verb fly, and confuses tonight and flight.
  8. C (A) uses the verb leave in the past, but refers to a different subject. (B) confuses relieved with leave.
  9. A (B) assumes the question Would you like to go to…? (C) refers to Australia, but answers a different question.
  10. C (A) is an illogical response. (B) refers to the past.
  11. B (A) answers a different question (How much did it cost?). (C) relates to days, but does not answer the question.
  12. C (A) cannot answer a choice question. (B) is an illogical response.
  13. C (A) relates leaving with finished. (B) confuses imports with report.
  14. A (B) uses the phrase feel like, not feel. (C) repeats today and good but answers a different question.
  15. B (A) repeats book, but with a different meaning. (C) repeats hotel, using next week as a distractor for soon.
  16. A (B) begins No, which answers the slightly different question Did Naomi say which…?. (C) answers a different question (When did Naomi leave college?).
  17. C (A) repeats money and back but answers a different question. (B) does not answer the question.
  18. B (A) repeats tell. (C) repeats help in a different context. Part 3 p123 Track 105
  19. B The woman answers the phone and offers to take a message. (A) confuses extension, line, and busy with technical problems. (C) and (D) cannot be inferred.
  20. A The woman tries Lucy Ward’s extension number and says the line is busy.
  21. B The woman asks the man if he’d like her to pass on a message. He says Yes please.
  22. A They are talking about moving filing cabinets, tables, and chairs to a different location in another building. (B) is a distractor for Maintenance Department. They are relocating their office, not moving home (C). (D) is not mentioned.
  23. C The man says Maybe we could hire a truck?
  24. A The woman says she will find out how much it’ll cost.
  25. B The man says he is working on this week’s sales report. They are both in an office. (A), (C) and (D) are incorrect.
  26. A The woman mentions going to lunch at a nearby restaurant, and the man says OK, that sounds good.
  27. B The man is working and says I’ll be along in a minute. We can assume he wants to keep working for a short while.
  28. C The words on board, passengers, take off, Customs and captain indicate this conversation takes place on a plane.
  29. A The man says they are waiting for two more passengers.
  30. C The woman says they can’t wait much longer, meaning they will wait for a short time.
  31. D The man complains about the desks covered in dust and paper all over the floor.
  32. D We can tell from his tone, and from what he says (I’m sick of it) that the man is angry.
  33. A The man says he wants to get rid of Andy, and advertise for a new cleaner.
  34. D The woman says they have won five big contracts recently. The man says they are starting to see the results of their hard work.
  35. A The woman says we got the Jefferson contract last week.
  36. D He says I’ll give them a small bonus. (B) and (C) are the woman’s suggestions. (A) misleads by referring to the contract they have won.
  37. C The man mentions the meeting room upstairs and later refers to impressing visitors. They can’t be in a hotel (A). The woman says she will go to a garden center (B). (A) confuses coffee table with coffee shop.
  38. A The man says we could put more lights in.
  39. C She will go to a garden center on Saturday to buy plants.
  40. B They are speaking about painting the ceilings and walls on the first floor rooms, and need to know the color. He man is a painter.
  41. A The man is annoyed when the woman says they haven’t received payment for last month’s work.
  42. B The woman suggests calling Joanne Masters at the start of the conversation. The man later says Tell her we need that money, implying the woman should call Ms. Masters.
  43. C They are talking about the dishes they are eating; fish and vegetable pizza. They are not in an office (A), or buying the food in a grocery store (B) or at a market (D). We can assume they are in a restaurant.
  44. D They can’t be housemates (A) or coworkers (C) as they do not see each other often. (B) is incorrect as the man asks How are things at work? They are most likely friends.
  45. A The man says We haven’t seen each other for ages.
  46. C The woman complains the roof leaks and the windows won’t close. They then discuss the cost of repairs.
  47. D She says she will ask someone to give us a price for the repair. She will get someone to take a look, not look herself (A) or fix the problem herself (B). She wants to get a quote, not send a quote (C).
  48. A The man says I hope it won’t cost too much. He is worried by the possible expense. Part 4 p125 Track 106
  49. D The speaker is giving the weather forecast on WTNC, which we can deduce from the context is a radio station.
  50. B He says Thursday will be wet but there will be sunny periods.
  51. C The forecast says there is a 70 percent chance of rain on Friday.
  52. B The phrases National Airlines flight, Gate 34, flight and final call all suggest this is in an airport.
  53. D The speaker advises Mr. Tomaselli to proceed directly to Gate 34 , as the flight is about to depart.
  54. A As the gate will close in five minutes, Mr. Tomaselli needs to get there quickly.
  55. A The speaker refers to your needs here at Ablewight Industries. He is presenting a sales pitch to executives at the company. (B) relates to the man’s field of expertise. (C) is his company. (D) refers to customers the man mentions.
  56. B The speaker is pitching to design and deliver a market research program.
  57. D The man says he and his team are experts at market research.
  58. A The words and phrases editorial and design departments, print room, books, and shipping room indicate this is a publishing company. (B), (C), and (D) are all mentioned as part of this.
  59. C The speaker mentions the articles that you all write. The audience is a group of reporters.
  60. B The woman says We’ll end in the museum.
  61. C The ad says the magazine features leading business experts (A), and asks people to subscribe, so is not aimed at retailers (B). (D) misleads by relating to International in the title of the magazine.
  62. C The magazine is delivered monthly to subscribers.
  63. A The ad says listeners need to subscribe for two years to get a free luxury ballpoint pen.
  64. A The man says that Ms. Wilkins has decided to step down from her position and will be moving to Marston Convention Center.
  65. B The speaker refers to the audience as department heads, and later mentions the name of the company as the Waterford Hotel.
  66. D He says that anyone interested should e-mail Human Resources.
  67. A This is an automated queuing system. The extension number is mentioned, but has not been dialed (B). (C) and (D) are incorrect.
  68. D The end of the message states your estimated wait time is eleven minutes.
  1. C The phrases make flight reservations and book hotel rooms indicate the company is a travel agency.
  2. A The speaker tells the people to clean the kitchen, tables, chairs, and cutlery before the inspection. (B) is not mentioned. The speaker says this is an emergency, but he is not speaking to emergency workers (C). Kitchen is mentioned, but not kitchen suppliers (D).
  3. B The man says the complaint was made by a member of the public last week.
  4. A The time is ten, and the inspectors are arriving at one.
  5. B The speaker is talking about this year’s charity fundraising event. The aim is to raise money (A), but she is talking about the event itself. (C) is not indicated. (D) refers to where the money raised will go.
  6. D As there are fourteen other companies, the total is fifteen.
  7. D The decision is made to hold a sponsored car wash.
  8. B The presenter says that Brinks Machinery will stop operations at the end of this month. (A) confuses shock to the town with earthquake. The President is not new to the job (C), and no job losses were announced previously so there can be no increase in job losses (D).
  9. C He is the President of Brinks Machinery.
  10. B Mr. Chadley blames cheap imports from overseas for the failure of the company.

READING

Grammar

Word Choice – Mini Test p

  1. C The structure used here is so + adjective (tired) + that.
  2. B We use no before nouns.
  3. A Would you like is followed by some before an uncountable noun.
  4. B We use enough (meaning sufficient) before countable nouns.
  5. D As time is uncountable, little is used after very.
  6. C Before the countable noun photographs, we use many.
  7. B Here most (meaning the majority of) is needed.
  8. A To refer to two related nouns, both is used.
  9. B Only never can complete this positive statement.
  10. D In this negative statement, any is used. Auxiliary Verbs – Mini Test p
  11. B Only cannot (can’t) is possible to express present inability.
  12. D We use have to as this is something that is necessary.
  13. A Could completes this polite request.
  14. D We use should have to express advice in the past.
  15. C Here we use didn’t have to (meaning it was not necessary).
  16. B To express negative possibility in the past we use could not have.
  17. D Here might expresses possibility.
  18. B To indicate present ability, we use can.
  19. A Occasionally, must is used to express present obligation.
  20. B Here, could completes the negative possibility. Comparatives and Superlatives – Mini Test p
  21. B The superlative of the two-syllable adjective serious is needed here.
  22. B The comparative using as…as completes this sentence.
  23. A The comparative more interested is used before than.
  24. C This option gives the complete comparative form of the three- syllable adjective expensive.
  25. A Only more modern can be followed by than.
  26. C To complete this comparative sentence, as…as is used.
  27. D Although separated, the comparative form here is worse than.
  28. A The superlative the least difficult is needed here.
  29. B After just (meaning equally) the structure as…as must be used.
  30. D The comparative form of this one-syllable adjective is nearer. Subject–Verb Agreement – Mini Test p
  31. A The subject is singular, so has completes this present perfect tense. 2. B Because the definite article is used, only is can complete the present continuous tense. 3. D Despite the plural managers, one of takes a singular subject. 4. C The object is plural (sports), but the subject is program, which is singular. 5. C Money takes a singular verb form, so was completes the past simple passive. 6. D The indefinite article is used here, so are completes the present continuous tense. 7. C The subject is Singapore, which takes the singular verb form. 8. B Time takes a singular verb form, so is completes the present simple tense. 9. B Nobody is a singular subject. 10. A Although the name of the restaurant uses the plural Chefs, it takes a singular subject. Pronouns – Mini Test p 1. D Only the reflexive pronoun correctly completes this sentence. 2. C The possessive pronoun is needed here. 3. A In this case we use the possessive adjective. 4. A Mr. Lee is the subject, so we need the subject pronoun. 5. C The possessive adjective is needed to complete this sentence. 6. A The subject is We, so we need the object pronoun. 7. B The object pronoun is needed here. 8. D The possessive adjective completes this sentence. 9. C In this case we use the reflexive pronoun. 10. B The possessive pronoun is needed here. Verb Forms and Tenses – Mini Test p 1. B For actions in progress in the past we use the past continuous. 2. D Here we need the present continuous for a planned future event. 3. A The action finished in the past, so the past simple is used. 4. C The present perfect is used for this unfinished action. 5. B For plans made earlier, we use to be followed by going to. 6. C Only the past continuous can fit here, for actions in progress in the past. 7. A The action is completed in the past, so we use the past simple. 8. C The present simple is needed to complete this sentence. 9. C With since, the present perfect is normally used. 10. D We use the present continuous for things happening around now. Gerunds and Infinitives – Mini Test p 1. C After forbid the infinitive with to is used. 2. A After mind we use the gerund. 3. C The gerund is needed here as the action (wasting money) started before. 4. B This is the subject of the sentence, so we need the gerund. 5. D After suggest we use the gerund. 6. B Here the gerund is used as the object of the sentence. 7. A The adjective + preposition good at takes the gerund. 8. C The gerund is used after regret as the action happened before. 9. B After make + object we use the infinite without to. 10. B The verb justify takes the gerund.

Vocabulary

Word Forms – Mini Test p

  1. B After the modal auxiliary must we use a verb here.
  2. C Only an adverb can describe the verb completed.
  3. B Here the countable noun is needed.
  4. C The noun is the only word that completes this sentence.
  5. C Only an adjective can describe the noun vehicle.
  6. D The noun way is described by an adjective.
  7. A The verb is used as an imperative.
  8. D The adverb is used because it describes the verb become.
  9. B After the modal auxiliary could the verb is used here.
  10. C Here the adjective describes the noun talk. Words with Similar Meanings – Mini Test p
  11. B The verb charge means ask someone to pay (+ for).
  12. D The verb reduce means make something smaller.
  1. D The noun needed here is growth (meaning expansion).
  2. B Only the superlative form completes this sentence.
  3. C These words look alike, but only description fits the sentence.
  4. A The phrase used here is to have trouble (meaning difficulty) +ing.
  5. A The adjective easy best fits the meaning of this sentence.
  6. C We use the preposition on with surfaces/floors, etc.
  7. A The phrasal verb to hold (someone) up means to delay.
  8. A Only the adjective irresponsible (meaning foolish) fits here.
  9. B The adjective is needed here.
  10. B The noun advantage (meaning benefit) fits here.
  11. A Planets can only be discovered.
  12. B This intensifying adverb emphasizes the adjective nervous.
  13. A This conjunction showing contrast fits here.
  14. B Only the possessive adjective completes this sentence.
  15. D The present simple tense is used with the stative verb want.
  16. C The verb make collocates with noun progress.
  17. A Here the present perfect tense is used with since.
  18. B The possessive pronoun is needed here.
  19. C Only in (meaning within a certain time) completes the sentence.
  20. C Here temporary (meaning for a limited time) fits here.
  21. A The extension was done by a contractor (i.e. building company).
  22. C The conjunction whether (meaning if) best fits this sentence.
  23. C The names of subjects (i.e. Economics) all take a singular verb form.
  24. D This modal auxiliary completes the polite request.
  25. D Only bill (meaning banknote) can be used here.
  26. B Here any is used as a determiner before the uncountable noun time.

PART 6 – Text Completion

Try It Out p

  1. B Here the verb to be is followed by the adjective pleased (to).
  2. B Used as a preposition, Without is followed by the noun help.
  3. C This modal auxiliary completes the phrase would like to.
  4. A The gerund is used after have fun (meaning enjoy).
  5. B The verb that collocates with results is announce.
  6. D Only the pronoun there can complete this sentence.
  7. C Here in (meaning within a certain time) is needed.
  8. A This conjunction fits the sentence, showing contrast.

Steps to Success p

Practice 1

  1. C The noun discounts (meaning price reductions) fits here.
  2. A We use the present simple tense in this case.
  3. C The preposition in is used for places and regions. Practice 2
  4. A The verb needed is remind (someone about something).
  5. D The simple present passive is used here.
  6. C The possessive adjective completes this sentence. Practice 3
  7. D Only this conjunction can be followed by this noun phrase.
  8. B The collocation used here is achieve goals.
  9. A The preposition on is used for dates. Practice 4
  10. B The conjunction and links these two positive statements.
  11. C The noun position means job in a company.
  12. D Only the noun search can complete this sentence. Practice 5
  13. B This adverb indicates the time has arrived earlier than expected.
  14. A Only returned has a meaning that makes sense here.
  15. A The noun choice (meaning selection) completes this sentence.

Review Test p

  1. A Only the adverb significantly can be followed by expand.
  2. C The present perfect is used with since.
    1. B Only this conjunction can complete the sentence.
    2. D We use such a before an adjective + noun.
    3. B The past simple is used because the action is completed.
    4. D Teeth whitening is a service that the company offers.
    5. A Here we need the conjunction so to indicate result.
    6. C The phrase needed here is take advantage of (meaning benefit from).
    7. A The present continuous passive completes this sentence.
    8. B The modal auxiliary should is used to indicate expectation.
    9. D We use the imperative after please to issue polite instructions.
    10. C Here the gerund is used after the phrase thank you (for+ing).

PART 7 – Reading Comprehension

Try It Out p

  1. D Dr. Stevens is responding to Mr. Katsuzawa’s letter asking why he was not selected for the course Computing for Beginners. She mentions there were sixteen spaces available, that they were evenly distributed between men and women, and that those selected were all unemployed, retired or have part-time jobs.
  2. B The course runs Monday through Friday from 10:00 A.M. until 1:00 P.M., making a total of fifteen hours.
  3. C Dr. Steven’s refers to Mr. Katsuzawa’s full-time job, and questions how he would manage to attend the course, which runs during normal office hours.
  4. B The e-mail begins by announcing the latest edition of “Work in Action”. Although (A) a survey is mentioned, this is not until the third paragraph so is not the main purpose. (C) and (D) are not mentioned.
  5. A Ms. Shaw works for Carpe Financial Management and writes about sources of financial support and funding. (B) is too general, (C) is incorrect as she is not publishing the magazine, and (D) is not mentioned.
  6. C The word confidential means private (i.e. answers will not be disclosed).
  7. B The e-mail begins Dear Customer. It is for anyone who receives the magazine “Work in Action.”
  8. A The ad states that the offer is available when you book online. (B) is not mentioned. (C) is incorrect because you must arrive (not book) on a red date. The offer is valid for new reservations, which does not mean new customers (D).
  9. C The ad mentions the very finest cuisine (A), that the hotel is ideally situated for exploring the city (B), and that it is a perfect venue for a … business meeting (D).
  10. A The ad states Full payment is due at the time of booking.
  11. D Mr. Hilper’s e-mail address indicates he works for Aranlo. He mentions Off Mart (A). He is writing to Central Stationery (C). He bought BestBrand printer cartridges (B).
  12. D Mr. Hilper’s e-mail mentions prices at Central Stationery and says they are in some cases more than double those charged at other stores. He goes on to give examples of high prices. (A), (B), and (C) are not mentioned.
  13. B Referring to OffMart, Mr. Hilper says I notice from their online shop…
  14. C In his e-mail, Mr. Hilper mentions one of the three filing cabinets cost $65. In his response, Mr. Prent says this third cabinet is the new model and is therefore more expensive.
  15. D Here, the word value means appreciate or consider important.
  16. B Ms. Arakawa mentions changes in the length of the group tour and also the departure time, and possibly lunch venue. (A) is incorrect as the tour hasn’t taken place yet. The tour is not for the staff to participate in (C) and it is already organized (D).
  17. B In her memo, Ms. Arakawa says the length of the group tours is no longer two hours. In the schedule, the group tours are shown as lasting 90 minutes (from 10:00 A.M. – 11:30 P.M.). They are thirty minutes shorter.
  18. D In the schedule, Mr. Walker is shown to take Group B on a tour of the Print Room in the afternoon.
  1. C In the memo, Ms. Arakawa ask Dr. Nicholson to finish his talk in 45 minutes. In the schedule, his talk starts at 3:00 P.M. He should therefore finish by 3:45 P.M.
  2. D Here, the word prompt means punctual or on time.

Improve Your Performance

Reading for Main Ideas Passage 1 p

  1. A recent conference. In the subject line, Ms. Williams refers to the conference last Tue and goes on to say it went extremely well.
  2. To thank an organizations’ staff. Ms. Williams begins I would like to say “many thanks” and finishes with please pass on our thanks to the staff at the Maybridge Center.
  3. A convention center. Ms. Williams refers to the Maybridge Center as a fantastic venue and says she looks forward to holding more events in your great facility. Passage 2 p
  4. Opposition to a proposed development. The article focuses on protests from residents at the plans to build a fast food restaurant and gas station.
  5. To report a community meeting. The article is about a special council meeting that took place last night.
  6. In a local newspaper. There is nothing to indicate this is from a college newsletter or science journal. It is a straightforward reporting piece in a local paper. Passage 3 p
  7. Application requirements. The fax concerns an application for a bank account, and is accompanied by an application form which Mr. Wong must complete and sign.
  8. To request information. Ms. Bertram wants to confirm some personal details from Mr. Wong.
  9. A potential investor. Mr. Wong is interested in a savings account. He wants to invest. Passage 4 p
  10. Cooperating in research. The letter mentions Dr. Gupta’s research interests, and discusses working together on a research project.
  11. To agree to work together. Mr. Harp says that the possibility of working with Dr. Gupta is an excellent idea.
  12. A scientific researcher. Dr. Gupta’s knowledge is in the field of fuel cells and renewable energy. Passage 5 p
  13. A job vacancy. The ad is headed Temporary Cashier and details the job requirements.
  14. People who want short-term employment. This is an ad for a full- time, but temporary (i.e. short-term) job.
  15. To support a job application. Ms. Jackson mentions her work experience and personal skills.
  16. She is working part-time. In her e-mail, Ms. Jackson says I work part-time as a cashier.
  17. It is very successful. The ad mentions it is Maine’s biggest seafood and fish restaurant. Mini Test – Reading for Main Ideas p
  18. B The article is mainly about Harney’s, which is described as a large electronic retailer. (A) is not mentioned. (C) is only mentioned in the final line. (D) is given as a reason for poor sales, but is not the main focus of the article.
  19. D The article focuses on the company’s fall in profits and poor performance. Five stores will close, but this is not the main focus (A). (B) is too general. Sales since the New Year (C) are referred to, but are not analyzed.
  20. C Only (C) is possible. (A), (B), and (D) do not match the style or content. 4. B The article mentions the company’s recovery plan is intended to reduce overheads (i.e. cut costs). There is no indication the company will leave Oregon (A), although five stores in Oregon will close. (C) and (D) are not mentioned. 5. A The company is clearly facing tough challenges. (B) is incorrect as profits are down. Although results are poor there is no indication the company will collapse (C). (D) is not mentioned. Reading for Details Passage 1 p 1. Ms. Gillingham is the Admissions Officer. 2. The courses take place on weekday evenings. 3. Mr. Courtney says he is especially interested in Skills for Work. 4. He asks for a brochure and a price list. 5. He says A friend told me. 6. Reduced fees are available for people with low incomes. Passage 2 p 1. In Melrose Hall. 2. Her lecture is Living Energy. 3. The Learning Brain lasts for two hours. 4. Sally Erskine the Academic Program Manager. 5. Her lecture finishes at 1:15 P.M. 6. The last paragraph states Only members of the university may attend. Passage 3 p 1. She got back yesterday. 2. They are waiting for payment for Phase Three. 3. She has been on a business trip to Seoul. 4. She is going to Beijing. 5. She will be back on Friday. 6. Rupert can call her at four o’clock this afternoon. Passage 4 p 1. The ad states We are specialists at removing viruses and spyware. 2. Any customers in Orlando who need emergency help. 3. The PC Total Care service. 4. At the bottom the ad says Call us to discuss your requirements. 5. The cost is $50 per hour. ($500 for ten hours.) 6. There is a free consultation with the web design team. Passage 5 p 1. The e-mail states that Mr. Sharpe is the Academic Advisor for the Dynamic Sales and Marketing course. 2. Five days. The dates are July 22–July 26. 3. Those who pass will receive a GeoTraining Certificate of Training award. 4. Ms. Burton mentions he can e-mail the course planning team. 5. He needs to pay $4000 (the course fee $3950 and $50 “priority registration” fee). 6. She is the Course Manager. 7. The location is given in the notice as The Hoover Building. 8. In her e-mail, Ms. Burton asks Mr. Jones to pay immediately. 9. At the bottom of the notice, the bank is given as Hanubo International. 10. Ms. Burton mentions this is necessary because he is registering very late. Mini Test – Reading for Details p 1. A The discount for this item is given as 12 percent in the fourth column. 2. C The Expected delivery times state around 5 days after the order is received. This is within a week. 3. C In the opening line, Mr. Bryson says Mr. Ames visited last Friday. 4. B The letter states that shipping is free for orders over £1000. The total order is above this amount. 5. C The Sub-Total (before VAT is added) is given as £3313.70.
  1. A She mentions train travel (B), receipts for all expenses (C) and stays in hotels (D). Only (A) is not mentioned.
  2. C The article is about creating successful relationships in business. (A), (B), and (D) are not mentioned.
  3. D Paragraph two ends Most of all, talk to your clients… (A), (B), and (C) are all mentioned, but are not the most important principles.
  4. C Step Two mentions Always do what you say you are going to do (A), Set yourself achievable goals (B) and assess your progress (D). Only (C) is not mentioned.
  5. A Here, the word fundamental means basic or essential.
  6. A Under Reasons, the customer writes As always, I was very pleased… which clearly indicates the customer has used the company before. (B), (C) and (D) cannot be inferred.
  7. B Here, the word unpleasant means disagreeable or objectionable.
  8. C The topic of this paragraph is the reception staff mentioned in the first line. Ivan is the translator (B). The writer is a customer (D). (A) is not mentioned.
  9. B The first line states Please find following our order….
  10. C The fax is sent on May 27, and Ms. Barkley wants the items by Friday, June 5th, which is the end of the following week.
  11. D The second paragraph mentions the order totals over a thousand dollars.
  12. D Sumiko asks readers to sponsor me if you can, and mentions that whatever you do give will go to a good cause.
  13. C In line 2 she states she is running in support of the American Stroke Association this year. (A) was the charity she supported last year. (B) is the company she works for. (D) is the company’s official charity.
  14. C She mentions This year, I am running… which suggest she has run the Boston Marathon before. She also refers to Last year, and says this year I want to do even better. She started training two months ago, so (D) cannot be correct. (A) and (B) cannot be inferred.
  15. B Under Course Objectives, the course is stated as being for medical staff at UHMC (United Health Medical Centers).
  16. A Next to the Sunday afternoon session Team Building is the comment see course folders for more details.
  17. C Here, the word competencies means skills or abilities.
  18. B They will stay at the Chatfield Institute which is a residential management training venue.
  19. D Gladys tells Sanjaya You MUST read the terms and conditions before applying. (A) and (C) are incorrect. (B) is not mentioned.
  20. C The chart indicates this year’s funding for Clothing to be $1500.
  21. A Here, the word confirm means check or verify.
  22. C Gladys mentions As this is your first time attending ETF, I think it is worth submitting an application. (A), (B), and (D) are not indicated.
  23. D In the e-mail, Gladys refers to the chart and tells Sanjaya your industry has been the hardest hit. The chart shows that funding for Construction fell most, from $2000 to $1000.
  24. D Mr. Ronson writes he is unable to continue and will be leaving his position. Staff cuts (A) and a pay raise (B) are covered, but are not the main purpose of the letter. (C) is not mentioned.
  25. B In her memo, Ms. Rebecci says he started in the beginning of April. The letter gives the date as September 30.
  26. A Ms. Rebecci states I have tried to bring us up to date and refers to the measures I have taken. (C) is incorrect as the memo states his coworkers will take over Mr. Ronson’s role. (B), and (D) cannot be inferred.
  27. B The memo states that a few months ago the company was in terrible shape. There have also been staff reductions. (A) is incorrect. (C), and (D) cannot be inferred.
  28. A Here, the word measures means steps or action.
  29. C Mr. Walliam is C.E.O. of Presoneat Assurance Co., Ltd which has offices in Sakura Building 1. (A) and (B) refer to the notice. (D) is not mentioned.
  30. C Mr. Walliam says some people continued working as normal (A), and that the fire alarm sounded at 5:20 P.M. (B). He also complains you did not give us a time (D). Only (C) is not mentioned.
    1. B The notice states you must leave the building immediately. (A) is not mentioned. (C) is incorrect and (D) refers to after they have left the building.
    2. C The notice states the fire alarm test will take place on Monday, August 17. Mr. Walliam refers to yesterday’s fire alarm test.
    3. A Here, the word scheduled means planned or timetabled.
    4. B The deadline is given as September 30. (A) is incorrect. (C) and (D) are not mentioned.
    5. A The position advertised is Maintenance Supervisor, and duties described include the supervision and management of operations. (B), (C), and (D) are not mentioned.
    6. D The ad states those with supervisory experience will be preferred. In the e-mail, Mr. Lyton writes I have never been a manager. He is used to working with computers (A) and has eight years’ experience (B). He qualified in 2001 (C).
    7. B In his e-mail, Mr. Lyton states the best way to reach me is on my cell phone.
    8. B Here, the word common means popular or widespread.

Reading Test

Part 5 p

  1. D Only this adjective has the correct meaning to fit the sentence.
  2. B We use much in negative sentences with the uncountable noun money.
  3. A This modal verb completes the deduction must have + past participle.
  4. D The object pronoun is needed to complete this sentence.
  5. B This pronoun is used in negative sentences.
  6. A The infinitive following want to is used here.
  7. B We use since to refer to a point of time in the past.
  8. C A fine is a sum of money to be paid as a penalty for breaking the law.
  9. A The subject information takes a singular verb form.
  10. D Only the reflexive pronoun correctly completes this sentence.
  11. C The collocation used here is pay attention.
  12. C The noun guarantee (meaning warranty) fits here.
  13. C The adverb is used here (meaning fortunately).
  14. C We use at with locations (e.g. factory gates).
  15. A The adverb that collocates with forbidden is strictly.
  16. B Here we need the present continuous for things happening around now.
  17. C We can only use any to complete this negative statement.
  18. D Here, the adjective is needed to describe it (i.e. the weather).
  19. B To describe the noun comments, the adjective critical is needed.
  20. B Only look into (meaning investigate) can fit here.
  21. D The conjunction unless (meaning if … not) is needed here.
  22. B Here, the phrase for external use means not to be swallowed.
  23. A The past participle provided by collocates best here.
  24. C Liz is the subject, so we need the subject pronoun.
  25. B The only adverb that collocates with coming is definitely.
  26. A The structure used here is no alternative but (to + infinitive).
  27. C Only the conjunction showing contrast Although fits here.
  28. B Only the verb insist is followed by the preposition on.
  29. D The conjunction or correctly connects the two actions.
  30. A The adjective (meaning logical) describes the noun decision.
  31. D The comparative structure used here is more … than.
  32. B The verb allow someone to do something is used here.
  33. C Only in (meaning within a certain time) fits here.
  34. C The present perfect is used for this recently completed action.
  35. A The verb is used as a negative imperative.
  36. B We use decide between when there are two alternatives.
  37. D The past participle ending in -ed is used here.
  38. A Only the noun competes this sentence.
  39. C The verb to bring (someone something) fits here.
  40. B The collocation used here is casually dressed. Part 6 p
  41. B The adjective scheduled (meaning planned) fits here.
  42. C The structure used here is most of + noun.
  1. A The collocation used here is out of action (meaning not working).
  2. D If something is done in advance it is done ahead of time.
  3. B The future simple is used to indicate future intention.
  4. C The modal auxiliary should is used to indicate expectation.
  5. C The verb is used as an imperative.
  6. A The verb used here is welcome (someone + to …).
  7. B Of these nouns, only obstacles can be used with the verb overcome.
  8. D Only this pronoun, meaning No matter what, fits here.
  9. C These words have a similar meaning, but only numerous can come before companies.
  10. A Here, the infinitive is used after the verb help. Part 7 p
  11. B This itinerary is to attract people to sign up for a short tour of famous places in the UK. It is not aimed at tour operators (A). Tourist attractions (C) and historic sites (D) are mentioned, but only as details of the tour.
  12. D Walking tours of (A) and (B) are mentioned, and a coach tour of (C).
  13. C The Cotswolds is described as picturesque (meaning beautiful) and an Area of Outstanding Natural Beauty. (B) refers to the Lake District. (C) refers to York. Only one castle, Edinburgh Castle, is mentioned (A).
  14. B The e-mail begins by giving confirmation of your participation at the fair. (A), (C) and (D) are mentioned only as supporting details.
  15. A The e-mail states Contact details can be found on the order confirmation, which is attached.
  16. C She is told she can edit your registration data online. (A), (B) and (C) are mentioned, but are not relevant.
  17. D Here, the word assigned means allocated or given.
  18. C Mr. Hanrahan’s account has exceeded its credit limit. (A) refers to the overdraft charge incurred. There is no offer to raise (B) or correct (C) his credit limit.
  19. B He has an account with the bank, so he is a customer. Mr. Marshall is an employee (A). (C) and (D) are not directly mentioned.
  20. A He is asked to Please remit the amount outstanding. Only if this is not possible should he visit his local branch, not call (B) and speak to a client services manager for assistance (D). Further charges (C) will be applied if he does not resolve the situation.
  21. C Francis says he can’t conduct the performance review next Monday, and asks Suzanna to check your diary for an alternative date. He says he can’t reschedule the doctor’s appointment (A). (B) and (C) relate to the purpose of the meeting.
  22. D She is asked to list her goals (A), successes (B), and anything you want to ask about (C). Only (D) is not mentioned.
  23. B Here, the word productive means constructive or useful.
  24. C Francis is conducting Suzanna’s performance review, so we can assume he is her manager. (A), (B), and (D) cannot be inferred.
  25. D A consultant will visit your home (A), measure the dimensions and design a perfect layout (B), and discuss your requirements (C). Only (D) is not mentioned as free of charge.
  26. A The ad mentions visiting customers and installing kitchens at their homes. (B) and (D) are not mentioned. (C) refers to the company’s own consultants.
  27. D The ad states that One call is all it takes. The company designs, builds and installs kitchens. (A) and (B) are not indicated. (C) is incorrect as the company was Established in 2003.
  28. A The bottom of the ad states Order this month and pay nothing for one year.
  29. B The notice warns patients that doctors will video some of their consultations, but consent is voluntary for patients that participate. (A) and (C) are mentioned, but are not the intended audience. (C) is not mentioned.
  30. A No training was possible because of limited space at the previous premises.
    1. C The memo states that the purchasing policy is not being followed consistently and explains the correct procedure. (A) is not mentioned. (B) and (D) are mentioned, but not as the main purpose.
    2. B Orders over $300 need to be authorized by the Line Manager. (A) is too general. (C) is only necessary for orders over $1000. (D) refers to recommended suppliers.
    3. D Here, the word consortium means group or association.
    4. C The article focuses on one job. This is not a job vacancy (A), not does it mention several careers (B). (D) is mentioned but only at the very end.
    5. D Familiarity with advanced computer programs (A), working in an organized and systematic way (B), and excellent analytical skills to identify and fix problems (C) are all mentioned. Only (D) is not mentioned.
    6. B The letter details various course requirements. (A) is not mentioned. (C) and (D) are mentioned, but not extensively.
    7. B While he is away Mr. Halson asks Mrs. Petronelli to please e-mail me if necessary.
    8. C The first assignment is due March 4. The course begins on April 5. (A) and (B) are not mentioned. (D) cannot be inferred.
    9. B The ad mentions that BizPrint products are of the highest quality (A), and help you create a professional image (C). Full- color printing (D) is mentioned. Only the size of the cards (B) is not mentioned.
    10. D He complains details of the offer were not explained in the advertisement, that there was no mention of the $9.99 shipping fee and that he was not aware BizPrint would appear on the back of the cards. (A), (B) and (C) cannot be inferred.
    11. C He says this was not in the package I was sent.
    12. C Here, the word matters means things (i.e. problems).
    13. B The invoice of $19.99 (C) minus the $9.99 shipping fee (A) gives the regular price of the 250 business cards.
    14. B Mr. Dewitt’s e-mail asks for clarification of several issues; the number of accounts, meter readings, paperless billing and his tariff. He has already swapped energy accounts (A), and requested paperless billing (C). He does not want to change his tariff (D).
    15. B His e-mail begins by stating he applied Three weeks ago. The date is given as June 5, so he applied around the middle of May.
    16. B Mr. Dewitt mentions he can see three accounts, but in Mr. Hull’s reply three days later he says the electricity order has already disappeared.
    17. D Here, the word assured means promised or informed.
    18. A Mr. Dewitt does not need to take any action. He should only contact Mr. Hull (B) if necessary. (C) is not mentioned. Mr. Hull will take care of (D).
    19. A The notice concerns the launch of Staff Forum, and also the Staff Opinion Survey. Both initiatives aim to help staff express their views. (B) only relates to the Survey. (C) is mentioned but is not the main focus of the notice. (D) is incorrect because the survey has not been done yet.
    20. C Ms. Northton writes she wants to promote a more pro-active response. This is why she is offering a bonus to anyone who does volunteer, not just (A). (B) is incorrect. (D) is not mentioned.
    21. C The memo is to All Department Heads. The notice refers to six staff, one from each department. Therefore there are six departments.
    22. A Here, the word address means deal with or sort out.
    23. A In her memo, Ms. Northton writes that after three weeks I have yet to receive any expressions of interest in the two initiatives. We can infer she is disappointed at the response. She only asks the department heads to pass on news of the financial incentives, rather than take more responsibility (C). (B) and (D) cannot be inferred.
    24. D The article gives advice on Choosing the right policy, rather than recommending travel insurance (A). (B) is too general. There is no comparison of products (C).